LSAT and Law School Admissions Forum

Get expert LSAT preparation and law school admissions advice from PowerScore Test Preparation.

 Administrator
PowerScore Staff
  • PowerScore Staff
  • Posts: 8916
  • Joined: Feb 02, 2011
|
#22979
Complete Question Explanation

Assumption. The correct answer choice is (A)

The argument is that since testing for certain rare genetic flaws allows preventative treatment that is less expensive than treating diseases those flaws would otherwise lead to, widespread genetic screening would reduce the overall cost of healthcare.

You should notice that this is an evidence argument, and requires defense from potentially damaging evidence. Furthermore, the LSAT writers have given you a clue. The genetic flaws are rare, and to deal with that, the argument suggests widespread genetic testing. Actually, spending money on widespread testing for a rare problem might not be a very bright idea, if saving money is the goal. The defense will center around addressing that problem.

Answer choice (A): This is the correct answer choice. Applying the negation technique, we find that if the cost of treating the patients would not exceed the cost of widespread testing, the argument's conclusion that we would save money is wrong. This is an essential defense.

Answer choice (B): This is helpful information, but not essential. If only a few are preventable, we could still save money by addressing those before they become more costly, and could still save money by slowing the onset of the non-preventable diseases.

Answer choice (C): If the resources allocated to the treatment of those with genetic flaws increases, the argument is somewhat weakened, because that seems like an increase in cost, so this response is incorrect.

Answer choice (D): This response weakens the idea that there will be savings, and is incorrect. It addresses the possibility that there might be no savings because even though people know they have a likelihood of developing a disease, they might not be willing or able to invest in preventative measures. The argument requires defense against this response.

Answer choice (E): Since more common diseases are entirely irrelevant, and the argument did not concern making funds available in general to medicine, this response is incorrect.
 olafimihan.k
  • Posts: 25
  • Joined: Jul 04, 2017
|
#40259
Hello Powerscore,

I am having a heard time understanding how the correct answer choice is an assumption, it seems to be an answer choice that would more or less strengthen the conclusion. The reason why I chose answer choice B was because it seemed that the argument assumed that if MOST of the diseases linked to rare genetic flaws are preventable, then the conclusion that the OVERALL cost of healthcare would be reduced would apply. What am I not understanding ?

Thanks.
 Francis O'Rourke
PowerScore Staff
  • PowerScore Staff
  • Posts: 471
  • Joined: Mar 10, 2017
|
#40302
The author concluded that widespread screening would reduce the overall cost of health care, because it costs the system less to prevent a disease than to wait until it has occurred later. This is perfectly reasonable until you consider that
  1. the screenings cost money to perform
  2. a large section, if not the entire population would need to be screened while only a small minority have the rare genetic flaw
The author must assume that even given these factors, it would save the system money to screen for the genetic flaws. Answer choice (A) fills in this gap. If choice (A) were not true, then it would cost just as much or more to screen in advance as it would to treat later on.

Answer choice (B) states that the speaker must believe that most rare genetic diseases are preventable. However, the speaker only discussed some of the rare genetic flaws that are linked to certain diseases.
 chelsealit
  • Posts: 2
  • Joined: Oct 28, 2017
|
#41016
I'm a bit confused as to how answer choice A can be regarded as an assumption, given the fact that the author actually states in the stimulus "it costs the HCS less to prevent a disease than to treat it after it has occurred." Thus, if the author has already outright stated this fact, then how can it be an assumption missing from the argument?
 nicholaspavic
PowerScore Staff
  • PowerScore Staff
  • Posts: 271
  • Joined: Jun 12, 2017
|
#41089
Hi Chelsea,

Great question. This a classic Defender-Type Assumption question in that it eliminates ideas or assertions that would undermine the conclusion. And although your focus on the language of "it costs the HCS less to prevent a disease than to treat it after it has occurred" is correct, we also have to realize that a new piece of information is being added in the correct answer choice which is a classic technique of the LSAT in these types of questions.

Note how the correct answer is actually expanding on the idea of cost in a way that the original stimulus's statement did not. Because the blanket statement in the stimulus about prevention costing less than treatment is very broad. And it ignores the concept the cost of screening would have to be addded into the health-care system's pre-existing cost of treatment. So although the the stilmulus makes the argument that prevention costs less than treatment, is that really persuasive? In other words, try to imagine implementing the proposal. Because if we did, for a while at least we would be adding costs to the health care system by continuing with treatment, but adding in the additional, proposed screening costs. What does the author think is going to happen then? Is she assuming that the costs of screening will eventually lower the costs of treatment too? She most certainly is which leads her to the conclusion about the economics of the issue. But that temporary adding to the costs is definitely not mentioned in the stimulus which is what makes it an asssumption.

Thanks for the great question and I hope this helps! :-D

Get the most out of your LSAT Prep Plus subscription.

Analyze and track your performance with our Testing and Analytics Package.